A Question about Logical Consistency in Formal Logic

  • Thread starter Thread starter KiwiKid
  • Start date Start date
  • Tags Tags
    Logic
Click For Summary
The discussion centers on the logical consistency of the expression (\neg P \wedge \neg\neg\neg P) \vee P, with the poster expressing confusion about how to approach a formal proof without premises. They question whether the conclusion is logically consistent, highlighting that the inclusive OR allows for both \neg P and P to be true simultaneously, which seems contradictory. A suggestion is made to simplify the expression by reducing the triple negation, leading to (\neg P \wedge \neg P) \vee P. The conversation emphasizes the challenge of proving logical consistency in the absence of premises.
KiwiKid
Messages
38
Reaction score
0
First of all, I wasn't quite sure in which (sub)forum to post this, so if it doesn't quite fit, feel free to move it. I'm having a very hard time solving this one (or even seeing if it's logically consistent), and any help would be very much appreciated.

Homework Statement


Give a formal proof for the following conclusion:
(\neg P \wedge \neg\neg\neg P) \vee P

Homework Equations


There aren't any premises, so we're supposed to show that the conclusion is logically consistent.

The Attempt at a Solution


Beside the fact that I'm quite new to formal logic and don't really know how to start with this (having tried multiple things), there's something more important that's bothering me: I don't think this conclusion IS logically consistent.

You see, the logical OR is an inclusive-OR, which means that in the above case, you can have BOTH \neg P and P. But that makes no sense. Is that correct, or am I missing something? Can you still give a 'formal proof' for any such thing?
 
Physics news on Phys.org
KiwiKid said:
First of all, I wasn't quite sure in which (sub)forum to post this, so if it doesn't quite fit, feel free to move it. I'm having a very hard time solving this one (or even seeing if it's logically consistent), and any help would be very much appreciated.

I know there is a forum for logic and stuff, but not as a subset of homework help, so I'm not sure..

Homework Statement


Give a formal proof for the following conclusion:
(\neg P \wedge \neg\neg\neg P) \vee P

Homework Equations


There aren't any premises, so we're supposed to show that the conclusion is logically consistent.

The Attempt at a Solution


Beside the fact that I'm quite new to formal logic and don't really know how to start with this (having tried multiple things), there's something more important that's bothering me: I don't think this conclusion IS logically consistent.

You see, the logical OR is an inclusive-OR, which means that in the above case, you can have BOTH \neg P and P. But that makes no sense. Is that correct, or am I missing something? Can you still give a 'formal proof' for any such thing?

Well, I'm not sure about formal proof since you're only given P but...

(\neg P \wedge \neg\neg\neg P) \vee P

since there's a triple negation in the second P you can take two of them out, and you get...

(\neg P \wedge \neg P) \vee P

So that can be reduced...
 
Last edited:
Question: A clock's minute hand has length 4 and its hour hand has length 3. What is the distance between the tips at the moment when it is increasing most rapidly?(Putnam Exam Question) Answer: Making assumption that both the hands moves at constant angular velocities, the answer is ## \sqrt{7} .## But don't you think this assumption is somewhat doubtful and wrong?

Similar threads

Replies
2
Views
2K
  • · Replies 4 ·
Replies
4
Views
2K
  • · Replies 1 ·
Replies
1
Views
2K
  • · Replies 4 ·
Replies
4
Views
11K
  • · Replies 1 ·
Replies
1
Views
1K
  • · Replies 2 ·
Replies
2
Views
3K
Replies
3
Views
2K
  • · Replies 12 ·
Replies
12
Views
4K
  • · Replies 3 ·
Replies
3
Views
4K
Replies
20
Views
2K